LSAT and Law School Admissions Forum

Get expert LSAT preparation and law school admissions advice from PowerScore Test Preparation.

User avatar
 Dave Killoran
PowerScore Staff
  • PowerScore Staff
  • Posts: 5853
  • Joined: Mar 25, 2011
|
#27316
Complete Question Explanation

The correct answer choice is (C)

With the last rule suspended, the setup appears as follows:
Dec 97_M12_game#1_L9_explanations_Limited game#3_#5_diagram_1.png
This setup provides sufficient information to attack the questions.

Answer choice (A) is incorrect because from the third rule either S or T must be included in the main-dish recipe.

Answer choice (B) is incorrect because from the second rule either F or N must be included in the main-dish recipe.

Answer choice (C) is the correct answer choice.

Answer choice (D) is incorrect because from the third rule both S and T cannot be included in the main-dish recipe.

Answer choice (E) is incorrect because from the second rule both F and N cannot be included in the main-dish recipe.
You do not have the required permissions to view the files attached to this post.
User avatar
 LeslyLSATinLA
  • Posts: 16
  • Joined: Jan 30, 2023
|
#99358
So I was running short on time and simply guessed on this one and got it correct, can someone please explain why C is correct? my setup shows that by placing G and N separately, it would mean that S, P, L, N are in the main dish.
 Adam Tyson
PowerScore Staff
  • PowerScore Staff
  • Posts: 5153
  • Joined: Apr 14, 2011
|
#99372
S could be there, but so could T, because S and T are interchangeable in this game. Maybe S is in the Main Dish with T in the Appetizer, or maybe it's the other way around.

Your proposed solution could work, with F, G, and T in the Appetizer, but there are many other possible solutions, too. You have to go through the answers you are provided and eliminate those that break a rule, and only answer C does not break any rule.

Be careful about coming up with a hypothetical solution and then getting attached to it as if it is the only possible solution. Be a little more flexible about it and you'll be better able to see all the possibilities. The way we set up this question, we can't be sure where anything is, but we can be sure that the Main Dish has to have exactly one of S or T (one of them is in the group, the other is not, but there is no restriction on which one is which). We can also be sure that it has exactly one of F or N. Those two inferences are what we use to eliminate the wrong answers while remaining flexible with the other variables. Any answer that has both S and T must be out, as well as any answer that has neither S nor T. Same with F and N - if they are both in the answer, it's wrong, and if neither of them is there, it's wrong.

Get the most out of your LSAT Prep Plus subscription.

Analyze and track your performance with our Testing and Analytics Package.